Aviseringar
Rensa alla

Stämmer detta? (Mekanik uppgift)


Ämnesstartare

20150209_160703.jpg
Hej!

Jag undrar bara ifall jag har gjort rätt på den här uppgiften som har med mekanik att göra? Man skall finna vad för storlek som en puck har konstant k. Jag fick den till -0,313 något sådant. Tacksam för hjälp!

Om jag har gjort något fel?
Såhär ser uppgiften ut: llllll.png


   
Citera
Ämnesstartare

någon???


   
SvaraCitera
Ämnesstartare

ingen???


   
SvaraCitera
Ämnesstartare

särskriv inte


   
SvaraCitera

Uppgift

Formler

a=dv/dt
F=kv²
F=ma
v=ds/dt

Förutsättningar

m=0,2
s(t[1])=0
s(t[2])=35
t[1]=0
v(t[2])=v(t[1])/3

Lösning

F=kv²
F=ma
kv²=ma
(k/m)v²=a
a=(k/m)v²
a=dv/dt
(k/m)v²=dv/dt
v²=(m/k)dv/dt
v²-(m/k)dv/dt=0
v=1/(-(k/m)t+C[1])
v(t[1])=1/(-(k/m)t[1]+C[1])
t[1]=0
v(t[1])=1/(-(k/m)×0+C[1])
v(t[1])=1/(0+C[1])
v(t[1])=1/C[1]
1/v(t[1])=C[1]
C[1]=1/v(t[1])
v=1/(-(k/m)t+1/v(t[1]))
v(t[2])=1/(-(k/m)t[2]+1/v(t[1]))
v(t[2])=v(t[1])/3
1/(-(k/m)t[2]+1/v(t[1]))=v(t[1])/3
-(k/m)t[2]+1/v(t[1])=3/v(t[1])
-(k/m)t[2]=-1/v(t[1])+3/v(t[1])
-(k/m)t[2]=(-1+3)/v(t[1])
-(k/m)t[2]=2/v(t[1])
t[2]=-2m/(kv(t[1]))
v=ds/dt
ds/dt=v
s=∫((v)dt)
s=∫((1/(-(k/m)t+1/v(t[1])))dt)
s=-(m/k)ln|-(k/m)t+1/v(t[1])|+C[2]
s(t[1])=-(m/k)ln|-(k/m)t[1]+1/v(t[1])|+C[2]
s(t[1])=-(m/k)ln|-(k/m)×0+1/v(t[1])|+C[2]
s(t[1])=-(m/k)ln|0+1/v(t[1])|+C[2]
s(t[1])=-(m/k)ln|1/v(t[1])|+C[2]
s(t[1])=-(m/k)ln(|1|/|v(t[1])|)+C[2]
s(t[1])=-(m/k)ln(1/v(t[1]))+C[2]
s(t[1])=-(m/k)(ln(1)-ln(v(t[1])))+C[2]
s(t[1])=-(m/k)(0-ln(v(t[1])))+C[2]
s(t[1])=-(m/k)(-ln(v(t[1])))+C[2]
s(t[1])=(m/k)ln(v(t[1]))+C[2]
s(t[1])=0
(m/k)ln(v(t[1]))+C[2]=0
C[2]=-(m/k)ln(v(t[1]))+0
C[2]=-(m/k)ln(v(t[1]))
s=-(m/k)ln|-(k/m)t+1/v(t[1])|-(m/k)ln(v(t[1]))
s=-(m/k)(ln|-(k/m)t+1/v(t[1])|+ln(v(t[1])))
s=-(m/k)ln(|-(k/m)t+1/v(t[1])|v(t[1]))
s=-(m/k)ln(|-(k/m)t+1/v(t[1])||v(t[1])|)
s=-(m/k)ln|-(k/m)v(t[1])t+v(t[1])/v(t[1])|
s=-(m/k)ln|-(k/m)v(t[1])t+1|
s(t[2])=-(m/k)ln|-(k/m)v(t[1])t[2]+1|
t[2]=-2m/(kv(t[1]))
s(t[2])=-(m/k)ln|-(k/m)v(t[1])(-2m/(kv(t[1])))+1|
s(t[2])=-(m/k)ln|2+1|
s(t[2])=-(m/k)ln|3|
s(t[2])=-(m/k)ln(3)
s(t[2])=-ln(3)m/k
s(t[2])=35
-ln(3)m/k=35
-ln(3)m=35k
-ln(3)m/35=k
k=-ln(3)m/35
m=0,2
k=-ln(3)×0,2/35
k=-ln(3)/175
k≈-0,0063

Svar

k=-ln(3)/175
k≈-0,0063


   
SvaraCitera
Ämnesstartare

1239704331_gina-carano.gif


   
SvaraCitera
Ämnesstartare

dinkompis:
1239704331_gina-carano.gif

hon är så snygg..


   
SvaraCitera
Ämnesstartare

Inte mitt kompetensområde. 


   
SvaraCitera
Ämnesstartare

NiklasW:mitt kompetensområde. 

har du något


   
SvaraCitera

Önska jag kunde skriva ut läxor. Svårt när läxor sker uppvisningsvis o de man pluggar är monologer..


   
SvaraCitera